You hold a meter stick at one end with the same mass suspended at the opposite end. Rank the torque needed to keep the stick steady, from largest to smallest

Answers

Answer 1

The torque needed to keep the stick steady, ranked from largest to smallest, would be: highest when the suspended mass is at the far end of the stick, lower when the suspended mass is closer to the pivot point, and lowest when the suspended mass is at the pivot point itself.

To rank the torque needed to keep the stick steady from largest to smallest, we need to consider the factors that affect torque.

Torque is the rotational equivalent of force, and it depends on the distance between the pivot point (the end of the meter stick you are holding) and the point where the force is applied (the suspended mass), as well as the magnitude of the force.

In this scenario, the torque needed to keep the stick steady will be highest when the suspended mass is at the far end of the stick, i.e. as far away from the pivot point as possible.

This is because the greater the distance between the pivot point and the force, the more torque is required to counteract the force's rotational effect. Therefore, the torque needed to keep the stick steady will be highest when the suspended mass is at the end of the meter stick farthest away from the pivot point.

Conversely, the torque needed to keep the stick steady will be lowest when the suspended mass is at the pivot point itself, as there is no rotational effect to counteract in this scenario.

Therefore, the torque needed to keep the stick steady will be lowest when the suspended mass is at the end of the meter stick closest to the pivot point.

To know more about torque refer here

https://brainly.com/question/25708791#

#SPJ11


Related Questions

A man hikes 6. 6 km north along a straight path with an average velocity of 4. 2 km/h to the north. He rests at a bench for 15 min. Then, he hikes 3. 8 km north with an average velocity of 5. 1 km/h to the north. How long does the total hike last?.

Answers

The man hikes 6.6 km north with an average velocity of 4.2 km/h to the north. We can use the equation:

distance = velocity x time

to find the time it takes for him to complete the first part of the hike. Solving for time, we get:

time = distance / velocity

time = 6.6 km / 4.2 km/h

time = 1.57 hours

After resting at the bench for 15 minutes (or 0.25 hours), the man continues hiking 3.8 km north with an average velocity of 5.1 km/h to the north.

Again, we can use the same equation to find the time it takes for him to complete this part of the hike:

time = distance / velocity

time = 3.8 km / 5.1 km/h

time = 0.75 hours

To find the total time for the hike, we simply add the time for the first part of the hike, the rest, and the second part of the hike:

total time = 1.57 hours + 0.25 hours + 0.75 hours

total time = 2.57 hours

So, the total hike lasts for 2.57 hours. It's important to note that we assumed the man did not take any breaks during the second part of the hike, and that he continued hiking at a constant velocity. Additionally, we assumed that the path he took was a straight line.

However, in reality, the path may not be a straight line and the man may take breaks or adjust his velocity during the hike.

To know more about average velocity refer here

https://brainly.com/question/862972#

#SPJ11

the image below shows a photo taken with a built-in lens of a digital camera. the bottom photo is taken with the same camera, but with an additional wide-angle lens. which wave phenomenon best explains the distortion of the bottom image compared to the top? diffraction dispersion reflection polarization

Answers

The wave phenomenon that best explains the distortion of the bottom image compared to the top is distortion due to the optical effect of lens refraction.

When light passes through a lens, it undergoes refraction, causing it to bend and converge or diverge depending on the curvature of the lens surface. A wide-angle lens can cause more bending of light and wider coverage, resulting in a distorted image with a wider field of view. Diffraction is the bending of light waves around obstacles, while dispersion is the separation of light into its constituent colors. Reflection involves the bouncing of light off surfaces, and polarization is the alignment of light waves in a particular orientation.

To know more about wave phenomenon , here

brainly.com/question/15390698

#SPJ4

If your core temperature becomes colder, it is more difficult for oxygen to dissociate from hemoglobin at any po2.

Answers

When the core temperature of the body decreases, the metabolic rate also decreases, leading to less production of carbon dioxide.

This results in a decrease in the partial pressure of CO2 in the blood, which leads to an increase in blood pH.

A higher pH means that the blood becomes more alkaline, which makes it more difficult for oxygen to dissociate from hemoglobin.

The reason for this is that oxygen binds to hemoglobin more tightly at a higher pH, which is known as the Bohr effect.

Thus, as the core temperature becomes colder, the oxygen-hemoglobin dissociation curve shifts to the left, making it more difficult for oxygen to be released from hemoglobin and making it less available to the tissues that require it.

To know more about Bohr effect, refer here:

https://brainly.com/question/15970643#

#SPJ11

If the wavelength of an x-ray is
5.2 x 10^-11 m, what is its frequency?

Answers

The frequency of an x-ray with a wavelength of 5.2 x[tex]10^{11}[/tex] m is approximately 5.77 x [tex]10^{18}[/tex] Hz. The frequency (f) of an electromagnetic wave is related to its wavelength (λ) and speed (v) by the formula f = v/λ.

For x-rays, the speed of light is used, which is approximately 3 x [tex]10^{8}[/tex] m/s. Therefore, the frequency of an x-ray with a wavelength of 5.2 x [tex]10^{11}[/tex] m can be calculated as:

f = (3 x [tex]10^{8}[/tex] m/s) / (5.2 x [tex]10^{11}[/tex] m)

f ≈ 5.77 x [tex]10^{18}[/tex] Hz

Thus, the frequency of an x-ray with a wavelength of 5.2 x[tex]10^{11}[/tex] m is approximately 5.77 x [tex]10^{18}[/tex] Hz. This is an extremely high frequency, which is why x-rays are so powerful and can penetrate through dense materials like bone.

To know more about electromagnetic wave, refer here:

https://brainly.com/question/3101711#

#SPJ11

What is the spring constant of this spring?

Answers

Answer: D 400 N/m

Explanation:

A) Find the tension in each of the two ropes supporting a hammock if one is at an angle of θ1 = 10 ∘ above the horizontal and the other is at an angle of θ2 = 33 ∘ above the horizontal. The person sleeping in the hammock (unconcerned about tensions and ropes) has a mass of 66 kg . Express your answer using two significant figures.
T1= ? N
B) Express your answer using two significant figures.
T2= ? N

Answers

(a) The tension in the rope at an angle of 10° above the horizontal is approximately 798.5.4 N.

(b) The tension in the rope at an angle of 33° above the horizontal is approximately937.7 N.

What is the tension in the rope?

To find the tension in each rope, we can use the fact that the net force in the vertical direction must be zero since the person in the hammock is at rest. Let T1 and T2 be the tensions in the ropes, and let the x-axis point to the right and the y-axis point upward.

A) The forces acting on the person are their weight (mg) downward and the tensions T1 and T2 in the two ropes, which make angles θ1 and θ2 with the horizontal.

The vertical components of the tensions are T1sinθ1 and T2sinθ2, respectively, and the horizontal components are T1cosθ1 and T2cosθ2.

Therefore, we can write:

T1sinθ1 + T2sinθ2 = mg (vertical equilibrium)

T1cosθ1 = T2cosθ2 (horizontal equilibrium)

Solving for T1 and T2, we get:

T1 = (mgcosθ2) / (sinθ1cosθ2 + sinθ2cosθ1)

T1 = (66)(9.81 )(cos(33°)) / (sin(10°)cos(33°) + sin(33°)cos(10°))

T1 ≈ 798.5.4 N

B) Similarly, we can use the horizontal equilibrium equation to find T2:

T2 = T1cosθ1 / cosθ2 = (798.5 N)(cos(10°)) / cos(33°) ≈ 937.7 N

Learn more about tension on rope here: https://brainly.com/question/29466375

#SPJ1

26.0 g of mercury is heated from 28°c to 175°c, and absorbs 545 joules of heat in the process. calculate the specific heat capacity of mercury.

Answers

The specific heat capacity of mercury is approximately 0.142 J/g°C.

To calculate the specific heat capacity of mercury, we can use the formula:

Q = mcΔT

where Q is the heat absorbed (545 J), m is the mass of mercury (26.0 g), c is the specific heat capacity, and ΔT is the change in temperature (175°C - 28°C).

First, let's find ΔT:

ΔT = 175°C - 28°C = 147°C

Now we can rearrange the formula to solve for c:

c = Q / (mΔT)

Plugging in the values:

c = 545 J / (26.0 g × 147°C) = 545 J / 3822 g°C

c ≈ 0.142 J/g°C

So, the specific heat capacity of mercury is approximately 0.142 J/g°C.

To learn more about heat, refer below:

https://brainly.com/question/1429452

#SPJ11

Measuring the length from the lowest point of a spring-mass to the highest point, it is found to be 42 cm. What is the amplitude?


a. 42cm


b. 0. 42m


c. 84cm


d. 21cm

Answers

Therefore, the amplitude is: amplitude = 21 cm. So, the correct answer is d. 21cm.

A periodic variable's amplitude measures the change it undergoes throughout a single period. When measured against a standard value, a non-periodic signal's amplitude is its magnitude. There are several definitions of amplitude, all of which depend on how much the extreme values of the variable deviate from one another.

The amplitude of a spring-mass system is half the distance between the equilibrium position (the rest position of the mass) and the highest point of the oscillation (the crest).

Since the length from the lowest point to the highest point of the oscillation is given as 42 cm, the total displacement of the oscillation is 2 times the amplitude.

Therefore, the amplitude is:

amplitude = 42 cm / 2 = 21 cm

So, the correct answer is d. 21cm.

Learn more about amplitude Visit: brainly.com/question/3613222

#SPJ4

Explain how a balloon sticks to a wall.

What charge is the balloon?

What happens to the wall as you put the balloon near it? Why does this happen?

Include in your explanation the law of charges.

Be as detailed in your explanation as possible.

Answers

Answer:

We are assuming the balloon has been rubbed by a cloth, giving it extra negative charges.

After the balloon has been rubbed, it gains a negative charge because it gained some negative charges from the the cloth. This means there are more negative charges than positive ones to neutralize the effect, so the balloon gets a negative charge.

Due to the law of charges that states "Like charges repel each other; unlike charges attract," when the negatively charged balloon is brought near a wall, the wall's negative charges are repelled and pushed away from the balloon. Meanwhile, the positive charges in the wall are attracted to the balloon's negative charges. The strength of this attractive force is enough to keep the relatively light balloon attracted to the wall, which may sometimes keep it suspended in its place.

a single-turn current loop, carrying a current of 4.00 a, is in the shape of a right triangle with sides 50.0, 120, and 130 cm. the loop is in a uniform magnetic field of magnitude 75.0 mt whose direc- tion is parallel to the current in the 130 cm side of the loop. what is the magnitude of the magnetic force on (a) the 130 cm side, (b) the 50.0 cm side, and (c) the 120 cm side? (d) what is the magnitude of the net force on the loop?

Answers

The force on the 130 cm side is parallel to this combined force, the magnitude of the net force on the loop is 659.0 mN.

To solve this problem, we can use the formula for the magnetic force on a current-carrying wire in a magnetic field: F = I * L * B * sin(theta), where F is the force, I is the current, L is the length of the wire, B is the magnetic field strength, and theta is the angle between the wire and the magnetic field.

a) For the 130 cm side, the angle between the wire and the magnetic field is 0 degrees (since they are parallel), so sin(theta) = 0. Thus, the force on this side is F = I * L * B = 4.00 A * 1.30 m * 75.0 mT = 390.0 mN.

b) For the 50.0 cm side, the angle between the wire and the magnetic field is 90 degrees (since they are perpendicular), so sin(theta) = 1. Thus, the force on this side is F = I * L * B * sin(theta) = 4.00 A * 0.50 m * 75.0 mT * 1 = 150.0 mN.

c) For the 120 cm side, we can use the Pythagorean theorem to find that the angle between the wire and the magnetic field is approximately 36.9 degrees. Thus, sin(theta) = sin(36.9) = 0.6. The force on this side is F = I * L * B * sin(theta) = 4.00 A * 1.20 m * 75.0 mT * 0.6 = 216.0 mN.

d) To find the net force on the loop, we need to add up the forces on each side using vector addition. Since the forces on the 50.0 cm and 120 cm sides are perpendicular to each other, we can use the Pythagorean theorem to find their combined magnitude: sqrt((150.0 mN)^2 + (216.0 mN)^2) = 269.0 mN.

Since the forces on either side of the 130 cm are parallel to one another, we may add them:

269.0 mN + 390.0 mN = 659.0 mN.

The net force acting on the loop is 659.0 mN in size as a result.

To learn more about : magnitude

https://brainly.com/question/24468862

#SPJ11

Gravitational force between two bodies is 5N When they are placed at the distance of 1om.. How much gravitational force will be produced if they are kept at the distance of 20m.​

Answers

Answer:

F = 1.25 N

Explanation:

The equation to calculate Gravitational Force is

F = G (m1 . m2) / r^2

where G is gravitational constant, m1 and m2 are the mass of the 2 objects.

So, assuming that the G, m1, m2 is constant, the equation will be

F1 . [tex]r1^{2}[/tex]= F2 . [tex]r2^{2}[/tex]

Therefore,

F2 = F1 .  [tex]r1^{2}[/tex] /      [tex]r2^{2}[/tex]

And finally we just need to find F2 by inserting this value

F1 = 5N

r1 = 10m

r2 = 20m

I hope you can understand, let me know if you need more explanation.

Why is the wavelike nature of a moving baseball typically not observed?.

Answers

The wavelike nature of a moving baseball is typically not observed due to its relatively large mass and size in comparison to the extremely small scale of quantum mechanical effects, where wave-particle duality becomes significant.

Wave-particle duality is a fundamental concept in quantum mechanics, stating that particles like electrons can exhibit both particle-like and wave-like properties.

However, this behavior is most noticeable in extremely small objects, such as subatomic particles. The de Broglie wavelength is used to describe the wavelike nature of a particle and is given by the formula λ = h/(mv), where λ is the wavelength, h is Planck's constant, m is the mass of the particle, and v is its velocity.

For macroscopic objects like a baseball, the mass is large, making the de Broglie wavelength incredibly small. As the wavelength becomes smaller, the wavelike nature becomes less significant, and the object behaves more like a particle.

In the case of a moving baseball, the de Broglie wavelength is so small that the wavelike nature becomes essentially negligible and unobservable.

Furthermore, macroscopic objects like baseballs interact with their surroundings (e.g., air molecules) more frequently than subatomic particles.

This interaction, known as decoherence, reduces the visibility of quantum mechanical effects such as wave-particle duality.

In summary, the wavelike nature of a moving baseball is typically not observed due to its large mass and size, resulting in an extremely small de Broglie wavelength, and the frequent interaction with its surroundings, which reduces the visibility of quantum mechanical effects.

To know more about wavelike nature refer here

https://brainly.com/question/12751613#

#SPJ11

1. A boy moves on a skateboard at a constant velocity of 3 m-s-'. The


combined mass of the boy and the skateboard is 40 kg. He catches a bag of


flour of mass 5 kg that is thrown to him horizontally at 6 m-s-!. Determine


the velocity of the boy after catching the bag of flour. (2 m-s ' in his original


direction)

Answers

The velocity of the boy and skateboard after catching the bag of flour is 2.25 m/s in his original direction. We can use the conservation of momentum to solve this problem.

The initial momentum of the system (boy, skateboard, and flour) is:

p initial = (40 kg) x (3 m/s)

            = 120 kg·m/s

When the boy catches the bag of flour, there is no net external force on the system, so the total momentum remains constant.

Therefore, the final momentum of the system is also 120 kg·m/s. Let v be the final velocity of the boy and skateboard.

Then the momentum of the flour is:

p flour = (5 kg) x (6 m/s)

          = 30 kg·m/s

The total momentum of the boy and skateboard is:

p boy + skateboard = (40 kg) x (v)

So we can write the conservation of momentum equation as:

p initial = p boy + skateboard + p flour

Solving for v, we get:

v = (p initial - p flour) / (40 kg)

Plugging in the numbers, we get:

v = (120 kg·m/s - 30 kg·m/s) / (40 kg)

 = 2.25 m/s

Therefore, the velocity of the boy and skateboard after catching the bag of flour is 2.25 m/s in his original direction.

To know more about momentum refer here

brainly.com/question/12450698#

#SPJ11

A tuning fork has a 545 hz pitch. when a second fork is struck, beat notes occur
with a frequency of 6 hz. what are the two possible frequencies of the second fork?

Answers

The two possible frequencies of the second fork are 539 Hz and 551 Hz. To find the possible frequencies of the second fork, we can use the formula:

beat frequency = | frequency of fork 1 - frequency of fork 2 |

We know that the frequency of fork 1 is 545 Hz and the beat frequency is 6 Hz. So, we can set up two equations:

6 = |545 - frequency of fork 2|
6 = |frequency of fork 2 - 545|

To solve for the frequency of fork 2, we can isolate the absolute value and solve for both cases:

Case 1:
6 = 545 - frequency of fork 2
frequency of fork 2 = 539 Hz

Case 2:
6 = frequency of fork 2 - 545
frequency of fork 2 = 551 Hz

Therefore, the two possible frequencies of the second fork are 539 Hz and 551 Hz.

Know more about frequency here:

https://brainly.com/question/14316711

#SPJ11

which of the following travel at the same speed as light? check all that apply. which of the following travel at the same speed as light?check all that apply. x-rays. radar. microwaves. cell phone signals. radio waves. gamma rays. infrared radiation. ultrasonic waves.

Answers

The electromagnetic waves that travel at the same speed as light are x-rays, gamma rays, infrared radiation, radio waves, and microwaves.

The speed of light in a vacuum is a constant value, known as the speed of light, which is approximately 299,792,458 meters per second. All electromagnetic waves, including x-rays, gamma rays, infrared radiation, radio waves, and microwaves, travel at this speed in a vacuum.

Radar is an electromagnetic wave that is used for detecting and locating objects. It travels at a speed close to the speed of light but is not exactly the same. Ultrasonic waves, on the other hand, are sound waves that travel through a medium, such as air or water, and have a much lower speed than light.

Cell phone signals are a form of electromagnetic waves, but they do not travel at the same speed as light. Their speed is significantly lower and depends on various factors such as the distance from the transmitter, interference, and the type of carrier signal used.

In summary, only x-rays, gamma rays, infrared radiation, radio waves, and microwaves travel at the same speed as light, while radar, cell phone signals, and ultrasonic waves do not.

To learn more about speed click on,

https://brainly.com/question/31165460

#SPJ4

I need help on this one question in science



This image of a tiny fraction of the night sky was taken through a powerful telescope. Many of the objects seen in the image are galaxies similar to the Milky Way.




Telescopes have taken many images like this one, but of different fractions of the night sky. What do these images suggest?



A.


Each galaxy contains an equal number of stars.


B.


The Milky Way is the largest galaxy in the Universe.


C.


There are no other galaxies in the Universe.


D.


There are many other galaxies in the Universe

Answers

The statement that there are no other galaxies in the Universe is completely untrue. Science has shown us that there are countless galaxies in the Universe, each one containing billions of stars, planets, and other celestial bodies. The sheer size of the Universe alone suggests that there must be more galaxies out there.

Our own galaxy, the Milky Way, is just one of many, and we can observe other galaxies through telescopes and other instruments. In fact, astronomers estimate that there may be as many as 2 trillion galaxies in the observable Universe alone.

These galaxies come in many shapes and sizes, and they are spread out across the vast expanse of the Universe. Some are spiral galaxies like the Milky Way, while others are elliptical or irregular in shape. They all contain massive black holes, which play a crucial role in shaping the structure and evolution of the galaxies themselves.

Understanding the presence of other galaxies in the Universe is crucial to our understanding of the origins and evolution of the cosmos. Through ongoing scientific study, we continue to learn more about the structure, dynamics, and properties of these galaxies, shedding new light on the mysteries of the Universe.

To know more about galaxies  refer here

https://brainly.com/question/31361315#

#SPJ11

A crane in a marble quarry is mounted on the rock walls of the quarry and is supporting a 2000 kg slab of marble. The center of mass of the 900 kg boom is located one-third of the way from the pivot end of its 15-m length, and the cable supporting the boom is attached at 10. 0 m from the pivot end. What is the tension in the cable supporting the boom? g

Answers

A crane is lifting a 2000 kg marble slab in a quarry using a 15 m long boom that weighs 900 kg. The cable supporting the boom is attached 10.0 m from the pivot end and has a tension of 82184 N.

To find the tension in the cable supporting the boom, we can use the principle of torque equilibrium. This principle states that the sum of the torques acting on an object must be zero for the object to be in rotational equilibrium.

Here's a plan to solve the problem:

Hypothesis: The tension in the cable supporting the boom can be found using the principle of torque equilibrium.

Equipment/Techniques: We will need a calculator and knowledge of the formula for torque (torque = force x distance x sin(angle)).

Health and safety: This problem does not present any significant health and safety risks.

Data collection and analysis:

Quantities to be measured: We need to find the tension in the cable supporting the boom.

Number and range of measurements to be taken: We only need to calculate the tension in the cable once.

Equipment usage: We will use the formula for torque to calculate the tension in the cable.

Control variables: None.

Method for data collection and analysis:

Calculate the weight of the slab of marble:

[tex]W = mg = 2000\; kg \times 9.8 \;m/s^2 = 19600 N.[/tex]

Calculate the weight of the boom:

[tex]W = mg = 900 \;kg \times 9.8 \;m/s^2 = 8820 N.[/tex]

Calculate the torque due to the weight of the slab:

[tex]T1 = W1 \times d1 \times sin(\theta) = 19600 N \times 10 m \times sin(90) = 196000 Nm.[/tex]

Calculate the torque due to the weight of the boom:

[tex]T2 = W2 \times d2 \times sin(\theta) = 8820 N \times 5 m \times sin(60) = 24162 Nm.[/tex]

Calculate the torque due to the tension in the cable:

[tex]T3 = T \times d3 \times sin(\theta) = T \times 5 m \times sin(60) = 2.5T Nm.[/tex]

Apply the principle of torque equilibrium: T1 + T2 - T3 = 0.

Solving for T, we get T = (T1 + T2)/2.5 = (196000 Nm + 24162 Nm)/2.5 = 82184 N.

In conclusion, The tension in the cable supporting the boom is 82184 N.

To know more about tension refer here:

https://brainly.com/question/30033702#

#SPJ11

A sound source emits 20.0 w of acoustical power spread equally in all directions. the threshold of hearing is 1.0 × 10-12 w/m2. what is the sound intensity level 30.0 m from the source?

Answers

The sound intensity level 30.0 m from the source is approximately 92.5 dB.

To find the sound intensity level 30.0 m from the source, we need to follow these steps:

1. Calculate the sound intensity (I) at 30.0 m from the source:


Since the acoustical power (P) is spread equally in all directions, we can use the formula I = P / (4πr²),

where r is the distance from the source (30.0 m). So,

I = (20.0 W) / (4π × (30.0 m)²)

I = 20.0 / (4 × 3.14159 × 900)


I ≈ 1.77 × 10⁻³ W/m²

2. Calculate the sound intensity level (β) using the formula β = 10 × log10(I/I₀), where I₀ is the threshold of hearing (1.0 × 10⁻¹² W/m²). So,

β = 10 × log10((1.77 × 10⁻³ W/m²) / (1.0 × 10⁻¹² W/m²))

β ≈ 10 × log10(1.77 × 10⁹)


β ≈ 10 × (9.2477)
β ≈ 92.5 dB

The sound intensity level 30.0 m from the source is approximately 92.5 dB.

To know more about intensity level 30.0  refer here

brainly.com/question/11993021#

#SPJ11

What voltage is required to give the plates of a 270-pF capacitor a charge of 7. 3×10−9C?



Express your answer to two significant figures and include the appropriate units.



NEED HELP

Answers

27 V voltage is required to give the plates of a 270-pF capacitor a charge of 7. 3× [tex]10^{-9}[/tex] C.

The voltage (V) required to give the plates of a capacitor a charge (Q) can be calculated using the formula

V = Q/C

Where C is the capacitance of the capacitor.

In this case, the charge Q is given as 7.3 × [tex]10^{-9}[/tex] C and the capacitance C is given as 270 pF (pico-farads).

However, it is best to convert the capacitance to farads to ensure that the units are consistent

270 pF = 270 × [tex]10^{-12}[/tex] F

Now, substituting the values into the formula, we get

V = Q/C = (7. 3× [tex]10^{-9}[/tex] C) / (270 × [tex]10^{-12}[/tex] F) = 27 V

Therefore, the voltage required to give the plates of the 270-pF capacitor a charge of  7.3 × [tex]10^{-9}[/tex] C is 27 V (volts).

To know more about voltage here

https://brainly.com/question/31526382

#SPJ4

you see a burglar run by you. the police come by a few seconds later and ask you for help. the most useful quantity you could give them is the burglar'smultiple choice question.velocity.acceleration.time.speed.

Answers

The most useful quantity you could give the police in this situation is the burglar's: Velocity.



Explanation:
1. Velocity: It provides both the speed and direction of the burglar, which would be helpful for the police to track and catch them.
2. Acceleration: It refers to the rate of change in velocity, but it wouldn't be as helpful without knowing the initial velocity and direction.
3. Time: It's not particularly helpful in this situation, as it does not give any information about the burglar's movement.
4. Speed: While it gives the rate of movement, it lacks the direction, which is crucial for the police to know where the burglar is headed.

To Learn More About Velocity

https://brainly.com/question/80295?source=archive

SPJ11

If you double the kinetic energy of a nonrelativistic particle, how does its de Broglie wavelength change? The wavelength Choose your answer here by a factor of Type your answer here [factor answer should be given to one decimal place (ex. 1. 5)]

Answers

The de Broglie wavelength of a particle is inversely proportional to its momentum, so if the particle's kinetic energy is doubled. This means that the de Broglie wavelength will be halved, so the factor answer is 0.5.

What is wavelength?

Wavelength is the distance between two successive points of a propagating wave which have the same amplitude and phase. Wavelengths are typically measured in meters, centimeters, or nanometers, depending on the type of wave. Wavelengths range from radio waves, which have the longest wavelength, to gamma rays, which have the shortest wavelength. Waves with different wavelengths have different properties like speed, frequency, and energy. Wavelength is an important factor in determining the behavior of a wave, such as its reflection, refraction, interference, and diffraction. Wavelength also determines the type of electromagnetic radiation a wave produces, such as visible light, ultraviolet radiation, or infrared radiation. Wavelength is a fundamental property of waves and is used to describe the properties of light, sound, and other forms of energy.

To learn more about wavelength

https://brainly.com/question/10728818

#SPJ4

A puck slides on a frictionless table hitting a block. in which scenario does the puck exert the most force on the block?

Answers

The force exerted by the puck on the block depends on the rate of change of momentum during the collision.

To determine the scenario in which the puck exerts the most force on the block, we need to consider the principles of conservation of momentum.

The momentum of an object is defined as the product of its mass and velocity.

According to the law of conservation of momentum, the total momentum before the collision is equal to the total momentum after the collision, assuming no external forces are acting on the system.

Let's consider two scenarios:

Scenario 1: The puck approaches the block with a higher initial velocity.

Scenario 2: The puck approaches the block with a lower initial velocity.

In both scenarios, the mass of the puck and the block remains constant.

However, the difference lies in the initial velocity of the puck.

According to the conservation of momentum, the change in momentum of the puck must be equal and opposite to the change in momentum of the block.

If the initial momentum of the puck is greater in scenario 1 compared to scenario 2, the change in momentum will also be greater.

Since force is defined as the rate of change of momentum, a greater change in momentum implies a larger force.

Hence, in scenario 1 where the puck has a higher initial velocity, the puck will exert more force on the block during the collision.

To summarize, the puck exerts the most force on the block when it approaches the block with a higher initial velocity (scenario 1).

To know more about conservation of momentum refer here

https://brainly.com/question/24989124#

#SPJ11

on page 1 you had to draw the direction the earth moved without gravity in position 1 and 2. what was different about the path the earth took when there was not any gravity? * 5 points the earth moved in a straight line and did not move around the sun the orbit of the earth was faster the earth moved around the sun but travelled in a straight line the orbit of the earth stayed the same when you increased the mass of the sun, what happened to the gravity force? * 5 points it increased it stayed the same it decreased when you increased the distances, what happened to the gravity force? * 5 points it decreased it increased it stayed the same when you decreased the distances, what happened to the gravity force? * 5 points it increased it decreased it stayed the same when you increased the gravity, what happened to orbital speed? * 5 points it stayed the same it decreased it increased what are the 2 factors that affect the force of gravity? * 5 points the speed and size (mass) of an object the shape of the orbit and the distance between the objects nothing can change gravity because it is a natural force the size (mass) and distance between the objects

Answers

Changes in mass, distance, and gravity affect the force of gravity and orbital speed; the force of gravity is directly related to the size (mass) and distance between objects.

Changes in mass and distance affect the force of gravity and orbital speed: increasing mass or decreasing distance increases the force of gravity and orbital speed, while decreasing mass or increasing distance decreases them. The force of gravity is directly proportional to the product of the masses and inversely proportional to the square of the distance between them. The two factors that affect the force of gravity are the size (mass) and distance between the objects.

To know more about gravity, here

brainly.com/question/13224265

#SPJ4

--The complete question is, What are the effects of changes in mass, distance, and gravity on the force of gravity and orbital speed? What is the relationship between the force of gravity and the size and distance of the objects?--

a light sensor is based on a photodiode that requires a minimum photon energy of 1.65 ev to create mobile electrons. part a what is the longest wavelength of electromagnetic radiation that the sensor can detect?

Answers

The light sensor based on a photodiode with a minimum photon energy of 1.65 eV can detect electromagnetic radiation with a maximum wavelength of approximately 2.51 x 10⁻⁷ meters, corresponding to the infrared region of the spectrum.

To determine the longest wavelength of electromagnetic radiation that the sensor can detect, we need to convert the minimum photon energy of 1.65 eV into joules. Once we have the energy value in joules, we can use the equation that relates energy (E) and wavelength (λ):

E = hc/λ

where:
E is the energy of the photon,
h is Planck's constant (6.626 x 10⁻³⁴ J·s),
c is the speed of light in a vacuum (3 x 10⁸ m/s),
λ is the wavelength of the photon.

First, let's convert the minimum photon energy of 1.65 eV to joules. The conversion factor is 1 eV = 1.6 x 10⁻¹⁹ J.

Energy (E) = 1.65 eV * (1.6 x 10⁻¹⁹ J/eV)
= 2.64 x 10⁻¹⁹ J

Now, we can rearrange the equation to solve for the wavelength (λ):

λ = hc/E

Substituting the known values:

λ = (6.626 x 10⁻³⁴ J·s * 3 x 10^8 m/s) / (2.64 x 10⁻¹⁹ J)
≈ 2.51 x 10⁻⁷ m

Therefore, the longest wavelength of electromagnetic radiation that the sensor can detect is approximately 2.51 x 10⁻⁷ meters, which corresponds to the infrared region of the electromagnetic spectrum.

To learn more about electromagnetic radiation  click:

https://brainly.com/question/29646884

#SPJ12

Your camera's zoom lens has an adjustable focal length ranging from 80.0 to 205 mm. what is its range of powers (in d)

Answers

The range of powers for your camera's zoom lens is approximately 4.9 to 12.5 diopters. This means that the lens can focus on objects at different distances, providing flexibility and versatility when capturing images.

To find the range of powers of your camera's zoom lens, we need to first understand what the terms "focal length" and "power" mean.

Focal length (measured in millimeters) refers to the distance between the lens and the image sensor when the subject is in focus. In your case, the zoom lens has an adjustable focal length ranging from 80.0 to 205 mm.

Power (measured in diopters, or D) is a unit that describes the focusing ability of a lens. It is the inverse of the focal length (in meters). To find the power, we'll use the formula:

Power (D) = 1 / Focal Length (m)

Let's find the range of powers for your camera's zoom lens:

1. Convert the focal lengths to meters: 80.0 mm = 0.080 m, 205 mm = 0.205 m
2. Calculate the power for the minimum focal length: Power (D) = 1 / 0.080 m ≈ 12.5 D
3. Calculate the power for the maximum focal length: Power (D) = 1 / 0.205 m ≈ 4.9 D

For more about camera's zoom lens:

https://brainly.com/question/23841317

#SPJ11

Why was it important that dr. Jeff use a large ball to represent the sun a marble to represent the earth and a bead to represent the moon in his model

Answers

It was important for Dr. Jeff to use a large ball to represent the sun because the sun is much larger than the earth and the moon. Similarly, using a marble to represent the earth and a bead to represent the moon accurately represents their relative sizes in comparison to the sun. This helps to provide a visual representation that accurately depicts the sizes of the celestial bodies in question, which is important when teaching and understanding astronomical concepts.

Visit https://brainly.com/question/12075871 to learn more about the Solar System

#SPJ11

An iron Cub has each 15cm long at 20c. What will be :
1 the new surface of a face weather temperature rise to 80c
2 the volume of the same final temperature

Answers

a) The new surface area of one face will be 225.162 cm^2.

b) The volume of the iron cube at the final temperature of 80°C will be  3382.29 cm^3.

The thermal expansion of a solid material can be determined using the coefficient of linear expansion, which is a material property that relates the change in length to the change in temperature. For iron, the coefficient of linear expansion is approximately 1.2 x 10^-5 /°C.

a) To find the new surface area of a face when the temperature rises from 20°C to 80°C, we can use the formula:

ΔA = A_0 * α * ΔT

where ΔA is the change in surface area, A_0 is the initial surface area, α is the coefficient of linear expansion, and ΔT is the change in temperature.

For a cube with each side 15 cm long, the initial surface area of one face is 15 cm x 15 cm = 225 cm^2. The change in temperature is 80°C - 20°C = 60°C. Substituting these values and the coefficient of linear expansion for iron, we get:

ΔA = 225 cm^2 * 1.2 x 10^-5 /°C * 60°C = 0.162 cm^2

Therefore, the new surface area of one face will be 225 cm^2 + 0.162 cm^2 = 225.162 cm^2.

b) To find the volume of the iron cube at the final temperature of 80°C, we can use the formula:

ΔV = V₁ * β * ΔT

where ΔV is the change in volume, V₁ is the initial volume, β is the coefficient of volume expansion, and ΔT is the change in temperature.

For a cube with each side 15 cm long, the initial volume is 15 cm x 15 cm x 15 cm = 3375 cm^3. The coefficient of volume expansion for iron is approximately three times the coefficient of linear expansion, so we can use β = 3α.

Substituting these values and the change in temperature, we get:

ΔV = 3375 cm^3 * 3 * 1.2 x 10^-5 /°C * 60°C = 7.29 cm^3

Therefore, the volume of the iron cube at the final temperature of 80°C will be 3375 cm^3 + 7.29 cm^3 = 3382.29 cm^3.

To learn more about temperature click on,

https://brainly.com/question/12789820

#SPJ4

The microwave transmitters that we use have a frequency of about 10 ghz. What is the approximate wavelength?.

Answers

The approximate wavelength of a 10 GHz microwave transmitter is 3 centimeters.

The approximate wavelength of a microwave transmitter with a frequency of 10 GHz can be calculated using the formula:

wavelength = speed of light / frequency

where the speed of light is approximately 3.00 × 10^8 meters per second.

So, the wavelength of a 10 GHz microwave transmitter would be:

wavelength = 3.00 × 10^8 m/s / 10 × 10^9 Hz

wavelength = 0.03 meters or 3 centimeters

To know more about wavelength refer here

https://brainly.com/question/13533093#

#SPJ11

Robert and his younger brother Jake decide to go fishing in a nearby lake. Just before they cast off, they are both sitting at the back of the boat and the bow of the boat is touching the pier. Robert notices that they have left the fishing bait on the pier and asks Jake to go get the bait. Jake has a mass of 59. 5 kg and an arm reach of 50. 0 cm, Robert has a mass of 87. 5 kg, and the boat has a mass of 83. 0 kg and is 2. 70 m long. Determine the distance the boat moves away from the pier as Jake walks to the front of th

Answers

1. This problem involves the principle of conservation of momentum. Initially, the total momentum of the system is zero because they are all at rest.

When Jake starts walking toward the front of the boat, he exerts a force on the boat that causes it to move away from the pier.

To conserve momentum, the boat and Robert must move in the opposite direction to Jake's motion, so the total momentum of the system remains zero.

We can use the equation:

m1v1 + m2v2 = (m1 + m2)vf

where m1 and v1 are the mass and velocity of Jake and m2 and v2 are the mass and velocity of the boat and Robert before Jake starts walking. vf is the velocity of the boat and Robert after Jake reaches the front of the boat.

2. We can assume that Jake walks to the front of the boat in a straight line, which means that the boat moves in the opposite direction with the same speed.

We can also assume that the boat moves only a small distance compared to its length, so we can treat it as a point object.

Using the given values:

m1 = 59.5 kg

m2 = 87.5 kg + 83.0 kg = 170.5 kg

v1 = 0 m/s

v2 = 0 m/s

vf = -v1*m1/m2 = -0 m/s

Substituting these values into the equation and solving for vf, we get:

m1v1 + m2v2 = (m1 + m2)vf

0 + 0 = (59.5 kg + 170.5 kg)vf

vf = 0 m/s

This means that the boat and Robert do not move when Jake reaches the front of the boat. Therefore, the distance the boat moves away from the pier is zero.

To know more about momentum refer here

https://brainly.com/question/30677308#

#SPJ11

A water droplet falling in the atmosphere is spherical. assume that as the droplet passes through a cloud, it acquires mass at a rate proportional to ka where k is a constant (k>0) and a is its cross-sectional area. consider a droplet of initial radius r0 that enters a cloud with a velocity v0. assume no resistive force and show:

a. that the radius increases linearly with the time
b. that if r0 is negligibly small then the speed increases linearly with the time within the cloud.

Answers

A water droplet's radius will increase linearly with time if it acquires mass at a rate proportional to its cross-sectional area while passing through a cloud. This will cause its speed to also increase linearly with time within the cloud if its initial radius is very small.

a. As the water droplet falls through the cloud, it acquires mass at a rate proportional to its cross-sectional area. Since the droplet is initially spherical, its cross-sectional area is proportional to its radius squared, i.e., [tex]a \propto r^{2}[/tex]

Therefore, the rate of increase in mass of the droplet is proportional to k times r². By Newton's second law, the acceleration of the droplet is proportional to the net force acting on it, which is equal to the gravitational force minus the buoyant force.

Since there is no resistive force acting on the droplet, the buoyant force is proportional to the volume of the droplet, which is proportional to r³. Thus, the acceleration of the droplet is proportional to [tex](k \times r^2) - (constant \times r^3)[/tex]. Therefore, the radius of the droplet will increase linearly with time as it falls through the cloud.

b. If the initial radius of the droplet, r0, is negligibly small, then its initial mass and velocity will also be small. As it falls through the cloud, it will acquire mass at a rate proportional to its cross-sectional area, which is proportional to r². Therefore, the rate of increase in mass will be proportional to r².

The acceleration of the droplet will be proportional to the net force acting on it, which is equal to the gravitational force minus the buoyant force. Since the initial velocity of the droplet is small, the buoyant force will also be small, and can be neglected. Thus, the acceleration of the droplet will be proportional to r².

By Newton's second law, the velocity of the droplet will increase linearly with time, since the acceleration is proportional to r², which is proportional to the rate of increase in mass of the droplet.

In summary, if a water droplet falling in the atmosphere acquires mass at a rate proportional to its cross-sectional area as it passes through a cloud, then its radius will increase linearly with time, and if its initial radius is negligibly small, then its speed will increase linearly with time within the cloud.

To know more about mass refer here:

https://brainly.com/question/18064917#

#SPJ11

Other Questions
one problem with informal interviews to identify potential reniforcers is they are notO reliableO unconditionedO observationO behavior janet is retiring after working for a major department store for 20 years. the company offered her a flat monthly retirement benefit of $50 per year for each year of service. a. what was her monthly income in the first year after retirement? $83.33 b. what was her annual income for the first year of retirement? $50.00 c. after one year of retirement, she received a 1.54% cost of living adjustment to her monthly pension benefit. what was her new monthly benefit? Sam and jeremy have ages that are consecutive odd interferes. The product of their ages is 783. Which equation could be used to find Jeremys age, j, if he is the younger man? What is the public interest which the accounting profession serves? solve for x, neg x over 4 = 12 A 48 or B -3 or C 3 or D -48? A feeder neutral with a load of 400 A would be permitted the demand factor applied to _____ of the load?a. 100 Ab. 140 Ac. 200 A Suppose that you are a U. S. -based importer of goods from the United Kingdom. You expect the value of the pound to increase against the U. S. Dollar over the next 30 days. You will be making payment on a shipment of imported goods in 30 days and want to hedge your currency exposure. The U. S. Risk-free rate is 5. 0 percent, and the U. K. Risk-free rate is 4. 0 percent. These rates are expected to remain unchanged over the next month. The current spot rate is $2. 0. Required:a. Indicate whether you should use a long or short forward contract to hedge currency risk. B. Calculate the no-arbitrage price at which you could enter into a forward contract that expires in three months. C. Move forward 10 days. The spot rate is $1. 53 dollars per pound. Interest rates are unchanged. Calculate the value of your forward position Why did anheuser busch fires its entire marketing department? Your high independence only reveals the immeasurable distance between us Write a paragraph explaining how the advantages of a corporation led to the rise of big business and monopolies during the Industrial Revolution. Consider investors forming corporations and threats to competition. Does the cartoon below show any bias toward corporations and the changes during the Industrial revolution? Looking at the chart, do you think forming a corporation has any disadvantages? By looking at the cartoon, can you tell if big-business monopolies have any positives? 3.) Which image (Revere or Bufford) represents this account? Give details from the picture that prove your point. What were the causes for the empty treasure of France under Louis XIV? Assess any three causes.FIRST TO ANSWER WILL BE MARKED BRAINLIEST!!! during the closing phase, the project team should meet to group of answer choices review and evaluate performance of the project. get to know each other. help the team form to be able to perform efficiently on the project. control the schedule and the costs. find the surface area of the prism 6m 5m 8m what were european explorers searching for when they sailed to the americas? Seven days a year, Tiger Stadium becomes the fifth largest city in the state of Louisiana. Over 92,000 fans pack the stadium to watch the Tigers play. After the game, if the fans leave at a rate of 10% per minute, how long will it take before the stadium is half empty?1. Find the data using at least 10 numbers in the x column. 2. Create a scatter plot. Label the graph and show increments.3. Write an exponential equation.4. Interpret the meaning of the "a" and "b" in your function y=ab^x including the units.5. Find out how long it will take before the stadium is half empty and all the way empty. The equation m= 35g gives the distance in miles, m, that a car can travel using ggallons of gas. Q4. 1) If the car has gone 210 miles, how much gas was used?A4. 1) Write the amount of gas that was used to travel 210 miles here. Q4. 2) Write an equation that represents the inverse function: the gallons of gas as afunction of distance in miles. A4. 2) Write the equation of the inverse function here. Q4. 3) The equation m=hg gives the distance in miles, m, that a car cantravel using g gallons of gas, if it travels h miles per gallon. Write an equation that represents the inverse function: the gallons of gas asfunction of distance in miles for this new equation. A4. 3) Write the equation of the inverse function here. According to weber, in a modern, bureaucratized democracy what is one of the most important characteristics for a government official to have?. What was one possible advantage for Cherokees to resist relocation for so long?Write your response in two to three complete sentences 3d *5d*2=?Please what is 3d times 5d times 2